GI Pathology UWORLD Q&A

Lakukan tugas rumah & ujian kamu dengan baik sekarang menggunakan Quizwiz!

A 48-year-old man comes to the emergency department due to 3 days of progressively worsening retrosternal burning chest pain, dysphagia, and odynophagia. His medical problems include hypertension and end-stage renal disease. He underwent cadaveric renal allograft transplantation 8 months ago. The patient's temperature is 38.3 C (101 F), blood pressure is 130/80 mm Hg, and pulse is 94/min. His BMI is 31 kg/m2. The oral mucosa is pink and moist without ulcerations. Cardiopulmonary examination is normal. The abdomen is soft with mild epigastric tenderness. The patient's leukocyte count is 4,200/mm3. An esophagogastroduodenoscopy demonstrates linear, shallow ulcerations in the lower esophagus. Which of the following is most likely to be seen on esophageal biopsy?

Enlarged cells with intranuclear inclusions Cytomegalovirus esophagitis can occur in transplant patients and usually presents with odynophagia or dysphagia that can be accompanied by fever or burning chest pain. Endoscopy typically shows linear and shallow ulcerations in the lower esophagus, and histology usually shows enlarged cells with intranuclear inclusions.

A 48-year-old male is admitted to the hospital with severe epigastric abdominal pain and vomiting after an episode of binge-drinking. Four weeks later, he is found to have a palpable upper abdominal mass and a cystic lesion is visualized on computed tomography scanning (see image below, asterisk marks the lesion). The inner walls of the lesion are most likely lined with:

Fibrous and granulation tissue Pancreatic pseudocyst is a common complication of acute pancreatitis. It is a collection of fluid rich in enzymes and inflammatory debris. Its walls consist of granulation tissue and fibrosis. Unlike true cysts, pseudocysts are not lined by epithelium.

A 23-year-old man is hospitalized with acute pancreatitis that resolves rapidly on fasting. He has suffered 2 similar episodes during the past 2 years. A gallbladder ultrasound obtained during a previous episode was normal. The patient has no other medical problems and takes no medications. He does not consume alcohol. Which of the following tests should be considered during the workup of this patient?

Serum Triglycerides Eighty percent of acute pancreatitis cases are caused by gallstones and chronic alcoholism. Less common causes account for the other 20%. Inherited or acquired hypertriglyceridemia can cause acute pancreatitis if the serum level of triglycerides exceeds 1000 mg/dL.

A 35-year-old man is admitted to the hospital with a 2-day history of abdominal pain. The patient's condition deteriorates throughout the course of hospitalization, and he dies 5 days after admission. Autopsy reveals chalky white lesions in the mesentery. Histologic evaluation of the lesions reveals adipose cell destruction and calcium deposition. Which of the following is the most likely cause of this patient's autopsy findings?

acute pancreatitis, acute pancreatitis,In acute pancreatitis, release of lipase and other digestive enzymes causes fat necrosis with precipitation of insoluble calcium salts (ie, saponification), imparting a chalky white gross appearance. Microscopically, necrotic fat cells with calcium deposits are seen. In severe cases, fat necrosis can involve the mesentery, omentum, and other parts of the abdominal cavity.

Adenomatous and serrated polyps (ie, sessile serrated polyp, traditional serrated adenoma) are neoplastic polyps that have malignancy potential. Increasing polyp size is the most important risk factor for cancer; villous histology and high-grade dysplasia are additional risk factors

2cm villous adenomatous polyps A 50-year-old woman comes to the office for a preventive examination. She feels well and has no ongoing symptoms. Medical and family histories are unremarkable. The patient does not use tobacco or alcohol. Blood pressure is 132/76 mm Hg and pulse is 72/min. BMI is 25 kg/m2. Physical examination is normal. The patient is up to date on breast and cervical screening but has not undergone colon cancer screening. She is referred for colonoscopy, during which a single colon polyp is discovered. The polyp is removed and sent for histologic analysis. Which of the following pathologic findings is associated with the greatest risk of malignant transformation in this patient?

A 73-year-old woman is brought to the clinic by her niece due to a 10-day history of abdominal pain and loss of appetite. The patient does not like "going to the doctor" and has not been seen by a physician in many years. Frequent loose stools and loss of appetite have been present since the pain began. Family history is significant for colon cancer in a maternal aunt. Temperature is 37.8 C (100 F) and pulse is 98/min. Abdominal examination is significant for tenderness on deep palpation of the left lower quadrant, along with an appreciable mass. There is no inguinal lymphadenopathy. White blood cell count is 15,000/mm3. Which of the following is the most likely cause of the abdominal pain in this patient?

Acute diverticulitis Diverticulitis is characterized by inflammation of colonic diverticula. Patients have abdominal pain, nausea, vomiting, and changes in bowel habits. Low-grade fever and leukocytosis are common, and physical examination may demonstrate a tender mass in the left lower quadrant.

A 45-year-old woman is brought to the emergency department with severe abdominal pain, fever, and confusion. The patient's husband reports that, yesterday, she began to have worsening right-sided abdominal pain associated with nausea and several episodes of vomiting. This morning, she has been slow to respond and is confused about where she is. The patient has a history of depression and takes sertraline. Temperature is 39.5 C (103.1 F), blood pressure is 88/50 mm Hg, pulse is 104/min, and respirations are 26/min. She appears ill and confused, and the sclera are icteric. Abdominal examination demonstrates significant tenderness with guarding in the right upper quadrant. Which of the following is the most likely underlying cause of this patient's current condition?

Calculous obstruction of the common bile duct Acute cholangitis is a life-threatening infection characterized by fever, right upper quadrant pain, and jaundice (Charcot triad); hypotension and altered mental status (Reynold pentad) occur in severe cases. Acute cholangitis typically develops in the setting of biliary obstruction; common etiologies include gallstones, malignancy, and strictures.

A 26-year-old woman is evaluated for intermittent abdominal discomfort, diarrhea, and melena. The patient undergoes colonoscopy followed by total colectomy due to significant colonic abnormalities. Representative colon findings are shown in the image below. Her sister, who has the same biological parents and has no symptoms, also undergoes screening colonoscopy and has similar findings. If left untreated, which of the following is the most likely lifetime risk of colon cancer in the patient's sister?

Close to 100% Familial adenomatous polyposis (FAP) is an autosomal dominant disorder caused by germline mutation to the tumor suppressor gene adenomatous polyposis coli. Patients with FAP develop hundreds or thousands of colonic polyps; lifetime risk of colon cancer is close to 100%

A 53-year-old female treated with high-dose folic acid for anemia experiences a moderate increase in her hemoglobin level. She returns to clinic complaining of bilateral foot numbness and difficulty in walking. The latter symptoms are most likely related to which of the following?

Cobalamin deficiency Deficiency of vitamin B12 is associated with both megaloblastic anemia and neurologic dysfunction, while folate deficiency is associated with megaloblastic anemia alone. Moderate improvement in the hemoglobin level often occurs when a deficiency in vitamin B12 is treated with folate, or vice versa. Treatment of vitamin B12 deficiency with folate alone can actually worsen neurologic dysfunction.

A 32-year-old man with abdominal pain and periodic diarrhea is found to have heme-positive stools. Stool cultures show no growth of pathogenic organisms. Colonoscopy with biopsy of the colonic mucosa is performed. Light microscopy of the tissue sample is shown in the image below. This patient most likely suffers from which of the following disorders?

Crohn Disease Noncaseating granulomas are a hallmark of Crohn disease and can help distinguish it from ulcerative colitis, which does not form granulomas. Histopathology in Crohn disease also demonstrates transmural inflammation, distortion of the normal architecture, and Paneth cell metaplasia.

A 28-year-old woman at 32 weeks gestation is seen in the office for a routine prenatal visit. The patient has had burning, squeezing pain in the middle of her chest that lasts for minutes to hours at a time and usually occurs after meals. The pain is occasionally accompanied by small amounts of sour-tasting material in the back of her throat. The patient does not have any abdominal pain, emesis, or visible blood in the stool. Her pregnancy has been complicated by gestational diabetes mellitus, which has been well controlled with nutritional therapy and exercise. She does not use alcohol, tobacco, or illicit drugs. Vital signs are normal. Fundal height is 32 cm and fetal heart tones are normal. Which of the following is the most likely cause of the patient's symptoms?

Decreased LES tone Pregnant women often develop gastroesophageal reflux disease (GERD) due to elevated levels of estrogen and progesterone, which relax the smooth muscle of the lower esophageal sphincter (LES). Later in pregnancy, GERD can also develop when the gravid uterus presses on the stomach and leads to an altered LES angle or increased gastric pressure.

A 10-year-old boy is brought to the hospital for difficulty swallowing.During dinner a few hours ago,a piece of chicken became stuck in the patient's mid-chest region. His parents say that he has previously had the sensation of food becoming stuck, but it has never lasted this long. The patient has vomited multiple times, but the chicken has not dislodged. Medical history is significant for eczema. He is drooling and visibly uncomfortable. An upper endoscopy is performed to remove the chicken from the esophagus, which exhibits multiple stacked, circular, ringlike indentations; linear furrowing; and scattered, small, whitish papules. Which of the following findings is most likely to be seen on biopsy of this patient's esophagus?

Eosinophilic infiltration of the esophageal mucosa Eosinophilic esophagitis is a Th2 cell−mediated disorder leading to eosinophilic infiltration of the esophageal mucosa. It occurs most commonly in males with a history of atopic conditions and typically presents with solid food dysphagia, reflux, and occasionally food impaction. Classic endoscopic findings include stacked, circular, ringlike indentations; linear furrowing; and scattered, small, whitish papules (ie, eosinophilic microabscesses).

A 43-year-old man is evaluated in the hospital for a medical illness. He has traveled to various countries on missionary trips and recently returned from South America. The patient has no prior medical history and takes no medications. His father has a history of alcohol dependence complicated by cirrhosis and portal hypertension. A liver biopsy is performed, and light microscopy of the tissue demonstrates spotty hepatocyte necrosis and inflammatory cell infiltration, as shown in the image below. Which of the following is the most likely initial clinical presentation of this patient?

Fever, anorexia, and dark-colored urine Acute hepatitis A is a self-limited infection that typically presents acutely with prodromal symptoms (eg, fever, malaise, anorexia, nausea/vomiting, right upper quadrant pain) followed by signs of cholestasis (eg, jaundice, pruritus, dark-colored urine, clay-colored stool).

Rats exposed to high concentrations of carbon tetrachloride suffer rapid and extensive liver damage. Light microscopic examination of affected liver specimens shows fatty change and hepatocyte necrosis. These changes are the result of:

Free radical injury The P450 microsomal oxidase system plays an important role in detoxification. In carbon tetrachloride poisoning, however, it produces free radicals that start a vicious cycle of hepatic injury.

A 40-year-old female presented to the physician for evaluation of worsening fatigue. She has no other medical problems. She does not use tobacco, alcohol or drugs. Vital signs are within normal limits. Physical examination is within normal limits. Laboratory evaluation shows an alkaline phosphatase level of 180 U/L. Which of the following should be checked next?

Gamma Glutamyl transpeptidase A moderately elevated alkaline phosphatase of unclear etiology should be followed up with γ-glutamyl transpeptidase

A 46-year-old woman comes to the office with persistent diarrhea, weight loss, and abdominal pain. Her diarrhea started several months ago. She has no fever, melena, or hematochezia. The patient has not traveled outside of the country and has not drunk from any untreated freshwater sources. She has had no recent hospitalizations or antibiotic use. Her sister has a history of celiac disease. Upper endoscopy reveals postbulbar duodenal and jejunal ulcers. This patient most likely has a tumor secreting which of the following molecules?

Gastrin Zollinger-Ellison syndrome is caused by gastrin-secreting tumors (gastrinomas) involving the small intestine or pancreas. Patients typically have peptic ulcers (often beyond the duodenal bulb), abdominal pain/acid reflux, and diarrhea. The condition is frequently associated with multiple endocrine neoplasia type 1.

A 53-year-old man with nocturnal cough undergoes an upper endoscopy with esophageal biopsy. Light microscopy of the tissue specimen shows elongation of the lamina propria papillae with several eosinophils and neutrophils scattered within the squamous epithelium. The patient takes metformin and enalapril for diabetes mellitus and hypertension, respectively. He is started on pantoprazole, and after 2 weeks his cough symptoms resolve. Which of the following was the most likely cause of his symptoms?

Gastroesophageal juntion incompetence Gastroesophageal reflux disease is caused primarily by gastroesophageal junction incompetence and can be associated with extraesophageal symptoms (eg, nocturnal cough) in the absence of heartburn ("silent GERD"). Acidic gastric contents irritate the esophageal mucosa, leading to characteristic histologic findings that include basal zone hyperplasia, elongation of the lamina propria papillae, and scattered eosinophils.

A 72-year-old woman comes to the office for follow-up of chronic abdominal pain. She has had several episodes of left lower quadrant pain requiring hospitalization. Despite treatment, her symptoms continue to recur. Elective surgery is performed and a segment of resected and opened sigmoid colon is shown below. Which of the following most likely contributed to this patient's condition?

High dietary intake of read meat and low fiber Colonic diverticulosis refers to multiple sac-like outpouchings within the sigmoid colon. Complications include diverticular bleeding and diverticulitis. Risk factors for diverticulosis include a diet high in red meat and fat and low in fiber, as well as obesity and physical inactivity.

A 45-year-old man comes to the emergency department due to sudden-onset vomiting and severe upper abdominal pain that radiates to his back. The patient's total bilirubin is 0.9 mg/dL, aspartate aminotransferase (AST) is 98 U/L, alanine aminotransferase (ALT) is 32 U/L, and alkaline phosphatase is 98 U/L. Serum lipase is markedly increased. Abdominal ultrasound reveals a normal gallbladder and common bile duct. The diagnosis of acute pancreatitis is made, and a specific etiology is established. Which of the following additional laboratory findings is most specific for the underlying cause of this patient's pancreatitis?

Mean corpuscular volume of 108 fl After gallstones, chronic alcohol use is the second most common cause of acute pancreatitis. Macrocytosis and an AST:ALT ratio >2 are indirect indicators of chronic alcohol consumption. Alcohol-related macrocytosis can occur independently of folate deficiency

A 3-week-old-girl in the neonatal intensive care unit develops abdominal distension, vomiting, and blood-streaked stools. She had previously tolerated formula feeds and had normal stool and urine output. She was born at 27 weeks gestation to an 18-year-old-mother. The pregnancy was complicated by premature rupture of membranes and preterm delivery. Abdominal x-ray shows thin curvilinear areas of lucency that parallel the bowel wall lumen. Which of the following is the most likely diagnosis?

Necrotizing enterocolitis Necrotizing enterocolitis is one of the most common gastrointestinal emergencies affecting newborns. It is characterized by bacterial invasion and ischemic necrosis of the bowel wall, and is associated with prematurity and initiation of enteral feeding. Abdominal x-ray showing pneumatosis intestinalis (ie, air in the bowel wall) confirms the diagnosis.

A 38-year-old woman comes to the hospital with 4 hours of abdominal pain, nausea, and vomiting. Her pain started in the periumbilical area and is now localized to the right lower quadrant. Temperature is 38.3 C (101 F). On examination, the patient has right lower quadrant tenderness with guarding. An appendectomy is performed, and light microscopy of the appendix shows infiltrating neutrophils in the muscularis propria, consistent with the diagnosis of acute appendicitis. Further evaluation reveals abnormal islands of small cells, as shown in the image below. These cells are most likely derived from which of the following sources?

Neluroendocrine cells Carcinoid tumors are composed of islands or sheets of uniform cells with eosinophilic cytoplasm and oval-to-round stippled nuclei. These tumors are often derived from neuroendocrine cells in the gastrointestinal tract. Appendiceal carcinoids typically have a benign course but may cause appendicitis or, rarely, carcinoid syndrome (eg, with liver metastasis).

A 62-year-old man comes to the office due to gnawing epigastric pain radiating to the back, which started 3 months ago and has become increasingly intense. He has also experienced a 10-kg (22-lb) weight loss and yellowing of the eyes. The patient has smoked 4 or 5 cigarettes a day for most of his life. Vital signs are within normal limits. The patient appears cachectic. Examination shows scleral icterus. The abdomen is tender to deep palpation. Abdominal CT scan is shown in the image below:

Pancreatic duct epithelium Pancreatic ductal adenocarcinoma is the most common pancreatic malignancy and typically presents with epigastric pain, weight loss, jaundice (more common if in the pancreatic head), or back pain (more common if in the pancreatic body/tail). It appears radiographically as an ill-defined hypoattenuating lesion within the pancreatic parenchyma. It is characterized microscopically by haphazardly arranged pleomorphic glandular cells surrounded by dense stromal fibrosis (eg, desmoplasia).

A 68-year-old woman comes to the emergency department due to shortness of breath, poor appetite, and abdominal distension for the past 3 months. Her symptoms have progressively worsened, and now she is unable to perform her daily activities. The patient has a history of coronary artery disease and underwent coronary artery bypass grafting 3 years ago. She also has myelodysplastic syndrome for which she receives intermittent blood transfusions. Physical examination shows jugular venous distension, abdominal distension, hepatomegaly, and lower extremity edema. A representative liver biopsy image is shown in the exhibit. Which of the following is the most likely cause of this patient's liver findings?

Passive hepatic congestion Congestive hepatopathy is a common complication of right-sided heart failure. Patchy hemorrhage and necrosis predominantly affect the centrilobular regions (zone 3) where hepatic congestion is most prominent. The centrilobular necrosis, combined with relatively normal-appearing periportal regions (zone 1), creates an overall heterogenous appearance sometimes referred to as "nutmeg liver."

A 5-month-old boy is being evaluated for poor weight gain since birth. He is noted to have bulky and greasy stools. A jejunal biopsy shows the findings seen in the slide below. Which of the following is the most likely diagnosis?

Abetalipoproteinemia is an inherited inability to synthesize apolipoprotein B, an important component of chylomicrons and very low-density lipoprotein. Lipids absorbed by the small intestine cannot be transported into the circulation and instead accumulate in the intestinal epithelium, resulting in enterocytes with clear or foamy cytoplasm.

A 68-year-old woman is evaluated after recurrent episodes of bright red blood per rectum. She has had no abdominal pain, nausea, vomiting, or hematemesis. Her medical conditions include hypertension and obesity. She is a smoker, has a sedentary lifestyle, and consumes processed food and red meat frequently. Abdominal examination shows no abnormalities. Colonoscopy reveals numerous mucosal outpouchings in the sigmoid colon, and sigmoid colectomy is performed. Histopathology of this patient's colonic lesions is most likely to reveal which of the following findings?

Absence of muscularis propia Colonic diverticula often involve the sigmoid colon and develop due to exaggerated contractions of colonic smooth muscle segments. This results in increased intraluminal pressure, causing outpouching of the mucosa and submucosa through the muscularis (false diverticula). Individuals (typically age >60) may be asymptomatic or have hematochezia or diverticulitis

A 58-year-old woman comes to the office due to a week of pain with swallowing. Her symptoms have progressively worsened, and now she cannot eat comfortably. The patient has a history of hypertension, osteoporosis, and primary biliary cholangitis progressing to cirrhosis. She underwent a liver transplant 2 years ago and is on immunosuppressive therapy. She does not use tobacco, alcohol, or illicit drugs. Vital signs are within normal limits. The oropharynx is normal in appearance. Cardiopulmonary examination is unremarkable. The patient has epigastric tenderness with no abdominal distension or guarding. Upper endoscopy shows several small esophageal ulcerations with distinct borderslocated in the middle and lower portions of the esophagus. A biopsy taken at the edge of an ulcer shows giant cells and intranuclear eosinophilic inclusions. Which of the following is the most appropriate treatment for this patient?

Acyclovir Herpes simplex virus esophagitis is most common in those with impaired cell-mediated immunity. Endoscopy typically reveals small vesicles and "punched-out" ulcers. Findings on biopsy usually include multinucleated giant cells and eosinophilic intranuclear inclusions (Cowdry type A inclusion bodies). Acyclovir is the treatment of choice.

A 49-year-old man comes to the office for evaluation of a dry cough that occurs mainly at night. His symptoms have been present on and off for several years, but he has not been previously evaluated. The patient also reports a frequent sore throat and occasional epigastric discomfort. He has tried a variety of antihistamines and over-the-counter cough medications without relief. Past medical history is notable for obesity and diet-controlled type 2 diabetes mellitus. He does not smoke or drink alcohol. Cardiopulmonary and abdominal examinations are unremarkable. Chest x-ray reveals no abnormalities, and pulmonary function testing is normal. The patient undergoes upper gastrointestinal endoscopy with mucosal biopsies. Microscopic findings on lower esophageal biopsy are shown in the image below. This patient is at highest risk of which of the following complications?

Adenocarcinoma Barrett esophagus is a metaplastic condition in which the normal squamous epithelium of the distal esophagus is replaced by intestinal-type columnar epithelium. It occurs most often in longstanding acid reflux and is associated with an increased risk of adenocarcinoma.

An 81-year-old woman is brought to the emergency department by a neighbor due to abdominal pain. The pain comes in waves and is associated with nausea and vomiting. The patient is confused and unable to provide additional information. Past medical history is unknown, but she comes with a bag containing medications used in type 2 diabetes mellitus, hypercholesterolemia, hypertension, and dementia. On examination, the patient has a low-grade fever and mild tachycardia. She appears in distress due to pain. Mucousmembranes are dry, and there is decreased skin turgor. Abdominal examination shows diffuse tenderness with high-pitched bowel sounds. Imaging of the abdomen reveals a complete small bowel obstruction. The patient undergoes laparotomy with extraction of a hard mass obstructing the ileocecal valve. The cholesterol content of the mass is 85%. Which of the following radiographic findings is most consistent with this patient's clinical presentation?

Air in biliary tree Gallstone ileus is a mechanical bowel obstruction caused when a large gallstone erodes into the intestinal lumen. Pneumobilia (air in the biliary tract) is a common finding.

A 62-year-old man comes to the office due to worsening dysphagia. Three months ago, the patient started having difficulty swallowing large bites of solid food, and now he can swallow only liquids and mashed food. He has lost 9.1 kg (20 lb) since the onset of his symptoms. Medical history is significant for type 2 diabetes mellitus, hypercholesterolemia, and hypertension. The patient recently reduced the doses of his diabetic medications due to weight loss and lower blood glucose levels. He has no recent travel outside of the United States. Vital signs are normal. BMI is 29 kg/m2. Physical examination is normal. A barium swallow study reveals an esophageal mass that significantly narrows the lumen. Endoscopic biopsy reveals moderately differentiated tumor cells with keratin nests and pearls. This patient's condition is most likely related to which of the following risk factors?

Alcohol consumption and tobacco smoking Major risk factors for esophageal squamous cell carcinoma include smoking, excessive alcohol consumption, and intake of foods containing N-nitroso compounds.

A 43-year-old man comes to the office due to yellowing of the skin over the past several days. He has had no fever or abdominal pain and has not traveled recently or received blood transfusions. The patient has also experienced increased shortness of breath with exercise over the past few months. He has no other medical conditions and takes no medications. The patient drinks 1 or 2 glasses of wine on social occasions and does not use tobacco or illicit drugs. His father died of liver cirrhosis attributed to alcohol use. On physical examination, expiration is prolonged and there are scattered bilateral wheezes. Laboratory results show total bilirubin of 4.1 mg/dL and alanine aminotransferase of 90 U/L. Viral hepatitis serology is negative. Chest x-ray shows hyperlucency of lung fields and flattening of the diaphragm. Abdominal ultrasound reveals increased liver span and echogenicity of the liver parenchyma but no masses. Which of the following is the most likely underlying cause of this patient's symptoms?

Alpha-1 antitrypsin deficiency Alpha-1 antitrypsin (AAT) is a serine protease inhibitor that regulates the activity of elastase in the lung. Inherited deficiency of AAT leads to alveolar destruction and panacinar emphysema; in addition, accumulation of improperly folded AAT proteins in hepatocytes can lead to liver dysfunction and cirrhosis in some patients.

A 58-year-old man with a history of chronic liver disease secondary to hepatitis C infection comes to the office due to progressive fatigue and abdominal discomfort. He has no other medical conditions, and a recent colonoscopy was normal. Physical examination shows ascites and jaundice. Ultrasonography of the abdomen reveals a solid mass within the liver parenchyma. Surgical resection of the liver lesion is performed. Microscopy reveals large cells with prominent nucleoli arranged in thickened plates as shown below. Which of the following serum markers would be most useful for monitoring this patient for disease recurrence?

Alpha-fetoprotein Alpha-fetoprotein is a serum tumor marker that is often moderately elevated in patients with chronic viral hepatitis. However, it can be strikingly elevated in those with hepatocellular carcinoma, and a sudden rise can be a sign that a patient with chronic liver disease is harboring hepatocellular carcinoma.

A 45-year-old man comes to the office due to a 1 year history of abdominal pain and greasy stools. He has also noticed a 7-kg (15.4-lb) weight loss over the past 6 months. Medical history is significant for several years of joint pain in his knees and wrists but is otherwise unremarkable. The patient does not use tobacco or alcohol. After an initial workup, a duodenal biopsy is performed. Microscopic examination shows numerous macrophages in the lamina propria filled with inclusions positive for periodic acid-Schiff stain. Which of the following is the most appropriate treatment for this patient?

Antibiotic Whipple disease, caused by the gram-positive actinomycete Tropheryma whipplei, is a rare systemic illness that often involves the small intestine, joints, and CNS. Classic intestinal histologic findings include foamy macrophages in the lamina propria filled with bacilli positive for periodic acid-Schiff stain. Treatment is with antibiotics.

A 65-year-old man is found to have iron deficiency anemia. He has had no cough, abdominal pain, melena, or change in bowel habits but reports anorexia and a 5-kg (11-lb) weight loss in the past 2 months. Cardiopulmonary and abdominal examinations are unremarkable. Rectal examination shows guaiac-positive brown stool. A 3-cm mass is found on colonoscopy. Biopsy shows pleomorphic cells with large, dark nuclei forming irregular, crowded glands, some of which contain mucus. Imaging studies reveal multiple mass lesions in the liver and lungs. This patient's neoplasm most likely originated from which of the following locations?

Ascending colon Colon adenocarcinoma is the most common gastrointestinal malignancy. Right-sided lesions are more likely to bleed and cause iron deficiency anemia; left-sided lesions tend to present with obstructing symptoms (eg, altered bowel habits, constipation, abdominal distension, nausea and vomiting).

A 38-year-old woman comes to the office due to jaundice, nausea, and abdominal discomfort. Vital signs are within normal limits. Scleral icterus and hepatomegaly are present. Laboratory results are as follows: Liver function studies Total bilirubin4.7 mg/dL Alkaline phosphatase110 U/L Aspartate aminotransferase (SGOT)791 U/L Alanine aminotransferase (SGPT)634 U/L Anti-smooth muscle antibody titers are elevated. Which of the following histologic findings is most likely to be seen on biopsy of this patient's liver?

Autoimmune hepatitis results from an immune response against hepatic antigens, leading to a lymphoplasmacytic infiltrate in the portal and periportal regions of the liver. Manifestations include fatigue, weight loss, nausea, and/or signs of acute hepatitis (eg, jaundice, abdominal discomfort). Characteristic laboratory abnormalities include a hepatocellular pattern of liver injury, anti-smooth muscle autoantibodies, and hypergammaglobulinemia.

A 45-year-old woman comes to the office due to several months of episodic right upper quadrant abdominal pain associated with nausea. The pain is often brought on by fatty meals and subsides in 1-2 hours. The patient has no fever, vomiting, diarrhea, melena, or bright red blood per rectum. Her BMI is 31.2 kg/m2. Physical examination is unremarkable. A cholecystokinin stimulation test is performed and shows slow and incomplete gallbladder emptying. This patient is most likely to have which of the following pathologic findings?

Biliary sludge (colic) The gallbladder functions to actively absorb water from bile. Gallbladder hypomotility causes the bile to become concentrated, promoting precipitation and accumulation of particulate material. This forms a viscous biliary sludge that can cause transient bile duct obstruction (biliary colic) and promote cholesterol gallstone formation.

A 32-year-old man comes to the office due to fever, jaundice, vomiting, and decreased appetite for the past 5 days. He recently returned home from South America and reports that a couple of his travel companions have had similar symptoms. The patient does not use alcohol, tobacco, or illicit drugs and has no relevant family history. Temperature is 38.8 C (101.8 F), blood pressure is 125/80 mm Hg, pulse is 85/min. Scleral icterus and hepatomegaly are present. Laboratory results are as follows: Complete blood count Hemoglobin14.9 g/dL Platelets389,000/mm3 Leukocytes15,000/mm3 Liver function studies Total bilirubin3.9 mg/dL Alkaline phosphatase185 U/L Aspartate aminotransferase (SGOT)2,410 U/L Alanine aminotransferase (SGPT)3,500 U/L Which of the following pathophysiologic events is most likely occurring in this patient?

CD8+ -T cell response against affefcted helatocytes Hepatitis A presents with fever, jaundice, hepatomegaly, leukocytosis, and aminotransferase levels >1,000 U/L. It triggers a robust CD8+ lymphocytic response to clear infected hepatocytes. The resulting hepatocellular damage is self-limited, with complete resolution within 2-3 months.

A 54-year-old man comes to the office due to increasing pain in the right upper quadrant, nausea, and unintentional weight loss. Vitals signs are within normal limits. BMI is 21 kg/m2. The patient appears cachectic and there is scleral icterus. The liver is palpable 3 cm below the right costal margin. No ascites is present. Laboratory studies reveal elevated bilirubin and alkaline phosphatase. Contrast CT scan of the abdomen demonstrates moderately dilated right and left intrahepatic bile ducts, with a vague area of delayed enhancement at their convergence.Biopsy of the lesion reveals columnar cells with hyperchromatic oval nuclei with prominent nucleoli arranged in glandular structures; mucin production with surrounding desmoplastic reaction is present. Which of the following is the most likely diagnosis?

Cholangiocarcinoma Cholangiocarcinoma is a malignancy of the bile duct epithelium that eventually obstructs biliary drainage, resulting in cholestasis (elevated alkaline phosphatase, hyperbilirubinemia) with jaundice, weight loss, pain in the right upper quadrant, and hepatomegaly. Imaging often demonstrates dilated bile ducts in the absence of an obstructive gallstone, and biopsy reveals an adenocarcinoma with columnar cells, mucin production, and a significant desmoplastic reaction.

A 46-year-old man comes to the office due to abdominal discomfort and diarrhea. The symptoms began 6 months ago, and he has now developed significant weight loss. The patient does not use tobacco or alcohol. Intestinal biopsy is performed, and a periodic acid-Schiff (PAS) stained section is shown in the image below: Which of the following is the most likely cause of this patient's symptoms?

Chronic bacterial infection Whipple disease is caused by a chronic infection with Tropheryma whipplei, a gram-positive bacterium that can affect multiple organ systems. It typically presents in middle-aged men with malabsorption and arthralgia. Intestinal biopsy characteristically shows foamy macrophages in the lamina propria filled with bacilli that are positive for periodic acid-Schiff stain.

A 44-year-old woman comes to the office due to indigestion. The patient says she often develops right upper quadrant abdominal discomfort and nausea with fatty meals, which subside spontaneously after several hours. She does not use tobacco, alcohol, or illicit drugs. The patient immigrated to the United States from Nepal 10 years ago. Abdominal ultrasound reveals numerous gallstones, and she undergoes elective laparoscopic cholecystectomy. The stones in her gallbladder have very low cholesterol content and appear small, dark, and spiculated. Which of the following conditions most likely predisposed this patient to gallstone formation?

Chronic hemolysis Black pigment stones arise from conditions that increase the amount of unconjugated bilirubin in bile, which promotes calcium bilirubinate precipitation. This may occur in the setting of chronic hemolysis (eg, sickle cell anemia, β-thalassemia, hereditary spherocytosis) and increased enterohepatic cycling of bilirubin (eg, ileal disease).

A 65-year-old man comes to the office due to progressive weight loss, jaundice, and anorexia over the last 3 months. His urine has been dark and his stools have been pale. The patient has no prior medical conditions and takes no medications. He has smoked a pack of cigarettes a day for 40 years. He drinks 2 cups of coffee every day and 1 or 2 glasses of wine on most nights. He used illicit drugs for 2 years when he was in his teens but has used none since. The patient has a sedentary lifestyle and frequently consumes red and processed meats. Vital signs are within normal limits. BMI is 28 kg/m2. Physical examination shows scleral icterus. The chest is clear to auscultation and percussion. Abdominal examination shows an enlarged but nontender gallbladder. There is no ascites. Which of the following is the strongest factor predisposing to this patient's current condition?

Cigarette Smoking Smoking is the most important environmental risk factor for pancreatic cancer. Other risk factors include advanced age, chronic pancreatitis, and genetic predisposition (eg, Peutz-Jeghers syndrome).

A 24-year-old woman comes to the office due to recurrent bloody diarrhea and right-sided abdominal pain. She has had these symptoms intermittently for several years but has not sought medical attention until now. Colonoscopy shows evidence of inflammation and dysplasia. The patient is started on appropriate medical therapy but her symptoms fail to respond adequately. She subsequently undergoes a colectomy of the involved region; findings are shown in the image. The intestinal wall appears thickened. Which of the following is the most likely diagnosis?

Crohn disease Crohn disease is an inflammatory bowel disease characterized by patchy inflammation that can occur throughout the entire gastrointestinal tract. Gross pathology demonstrates skip lesions, cobblestoning of the mucosa, bowel wall thickening, and creeping fat

A 26-year-old woman is evaluated for intermittent abdominal pain occurring over the last several years. The pain is crampy without radiation and graded 6/10 in intensity. She also has fluctuating diarrhea but has not seen blood in the stool. The patient has no nausea, vomiting, constipation, urinary frequency, dysuria, or vaginal symptoms. Her last menstrual period was 2 weeks ago. She does not use tobacco or alcohol. Temperature is 37.2 C (99 F), blood pressure is 115/70 mm Hg, and pulse is 90/min. On examination, the abdomen is tender without guarding or rebound. There is a draining fistula near her coccyx. Which of the following is the most likely diagnosis in this patient?

Crohn disease Crohn disease typically presents with the insidious onset of abdominal pain, diarrhea, and constitutional symptoms (eg, weight loss, fever). Patients are prone to developing fistulas/abscesses as the lesions affect the entire thickness of the bowel wall. Perianal disease (eg, skin tags, fissures) is also common.

A 28-year-old man with vague abdominal pain, low-grade fever, and diarrhea is treated with antibiotics without significant improvement in his symptoms. Several weeks after symptom onset, the patient develops a skin lesion over the abdomen. His temperature is 37.8 C (100 F), blood pressure is 120/70 mm Hg, and pulse is 88/min. On physical examination, the abdomen is mildly distended and tender to palpation. Bowel contents appear to be draining to the surface of the skin in the right lower abdominal quadrant. This patient most likely suffers from which of the following conditions?

Crohn disease Crohn disease typically presents with prolonged diarrhea and abdominal pain. Constitutional symptoms (eg, low-grade fever, fatigue), malabsorption, and weight loss are also common. Transmural inflammation of the bowel wall may result in the formation of fistulas (eg, enteroenteric, enterocutaneous), abscesses, and fibrotic strictures.

A 25-year-old man comes to the hospital due to worsening abdominal pain, distension, and nausea and vomiting for 3 days. He has a 4-year history of recurrent abdominal pain associated with diarrhea, low-grade fever, and easy fatigability. The symptoms usually occur after stress and resolve spontaneously in a few days. However, this time, the patient's symptoms persisted and worsened. He has no other medical problems and takes no medications on a regular basis. Temperature is 38.2 C (100.8 F), blood pressure is 110/70 mm Hg, pulse is 104/min, and respirations are 16/min. BMI is 19 kg/m2. Examination shows a tender mass in the right lower quadrant of the abdomen. Imaging is consistent with small-bowel obstruction. Laparotomy reveals that the abdominal mass is composed of inflamed small bowel, adherent and indurated mesentery, and enlarged abdominal lymph nodes. The affected region of the small bowel is resected. Which of the following is most likely to be seen on histologicexamination of this patient's intestine?

Crohn disease is characterized by patchy, transmural inflammation of the gastrointestinal tract. It can affect any part of the tract from the mouth to the anus. Complications include strictures (due to bowel wall edema, fibrosis, and thickening of the muscularis mucosae), fistulas (due to penetration of ulcers through the intestinal wall), and abscesses.

A 65-year-old woman comes to the office after finding blood in her stool. She is physically active and exercises regularly. She does not use tobacco or alcohol. She has no history of peptic ulcer disease. Her physical examination is unremarkable. Colonoscopy reveals several large adenomatous polyps with severe dysplasia in the sigmoid colon. She undergoes resection of the sigmoid colon, and her carcinoembryonic antigen level is normal. The patient is interested in decreasing her chances of developing new polyps and colon cancer. The physician discusses exercise, healthy weight, and dietary factors associated with reduction of colon cancer risk, including increased fiber intake and reduced consumption of refined sugars and fat. Which of the following enzymes could be inhibited with medication to further reduce this patient's risk of developing recurrent adenoma?

Cyclooxygenase-2 Adenomatous polyps contain dysplastic mucosa and are premalignant. Regular screening with timely excision of polyps is effective for prevention of colon adenocarcinoma. Studies have linked increased activity of cyclooxygenase-2 to some forms of colon adenocarcinoma and suggest that regular aspirin use decreases adenomatous polyp formation.

A 34-year-old man is evaluated for elevated liver aminotransferases. The patient has no chronic medical conditions but has a history of injection drug use.Family history is notable for liver cirrhosis in his mother. Ultrasonography-guided liver biopsy is performed. Histopathology demonstrates marked panlobular mononuclear cell infiltration that cross into adjacent lobules. Occasional intensely eosinophilic round bodies are seen scattered amongst the hepatic parenchyma. Which of the following is the most likely cause of thelatter histopathological finding in this patient?

Cytotoxic T cell- mediated apoptosis Intravenous drug use is a major risk factor for hepatitis B and C viral infection. Acute viral hepatitis is marked by panlobular inflammation and hepatocyte necrosis and ballooning. Cytotoxic T-cell-mediated signals also cause hepatocyte apoptosis with the formation of intensely eosinophilic Councilman bodies

A 21-year-old man comes to the office due to yellow discoloration of his eyesafter a hiking trip. The patient had a similar episode 2 years ago after fasting for a couple of days. Vital signs are within normal limits. Examination shows scleral icterus; there is no hepatosplenomegaly. Laboratory findings are as follows: Hemoglobin14.8 g/dL Reticulocyte count0.5% Albumin4.2 g/dL Total bilirubin2.8 mg/dL Direct bilirubin0.2 mg/dL Alkaline phosphatase90 U/L Aspartate aminotransferase (SGOT)20 U/L Alanine aminotransferase (SGPT)22 U/L Which of the following is the most likely cause of this patient's jaundice?

Decreased activity of UDP glucoronosyltransferase Gilbert syndrome is a benign disorder that presents with recurrent scleral icterus and jaundice, typically triggered by stress (eg, fasting, illness). Pathogenesis involves decreased activity of UDP glucuronosyltransferase (impaired bilirubin conjugation), and an isolated mild unconjugated hyperbilirubinemia is typical

A 32-year-old man with mild abdominal discomfort and anorexia admits consuming large amounts of alcohol recently. He undergoes a liver biopsy, which shows the light microscopy findings seen in the image below. Which of the following is the most likely cause of this patient's condition?

Decreased free fatty acid oxidation The pathogenesis of alcohol-induced hepatic steatosis appears related primarily to a decrease in free fatty acid oxidation secondary to excess NADH production by the 2 major alcohol metabolism enzymes, alcohol dehydrogenase and aldehyde dehydrogenase

A 54-year-old man comes to the office with abdominal pain, chronic diarrhea, and recent weight loss. He has had episodic abdominal pain in the past, but lately it has become persistent and worsens after eating. The patient does not use tobacco or illicit drugs but does consume alcohol regularly. An upright abdominal x-ray reveals calcifications in the epigastric area. Which of the following is the most likely cause of his diarrhea?

Digestive enzyme deficiency Diarrhea, weight loss, and epigastric calcifications in a patient with chronic alcohol use suggest chronic pancreatitis with resulting pancreatic exocrine insufficiency and malabsorption.

A 72-year-old woman comes to the emergency department due to bloody bowel movements. One hour ago, the patient had a sudden urge to defecate and passed a large amount of bright red blood mixed with stool. Several minutes later, she had another episode with a small amount of blood. The patient reports no nausea, vomiting, abdominal pain, diarrhea, or fever and never had such symptoms before. She is hospitalized but has no further bleeding. Physical examination, including a digital rectal examination, is unremarkable. A colonoscopy performed after bowel preparation reveals the findings in the exhibit. There are no other abnormalities. Which of following is the most likely source of this patient's bleeding?

Disruption of vasa recta Colonic diverticula form at weak points in the colon wall, typically in areas where the vasa recta penetrate through the smooth muscle. As diverticula enlarge, the vasa recta are exposed and become vulnerable to chronic injury, which can lead to intraluminal hemorrhage and painless hematochezia

An 18-year-old man is admitted to the hospital with cough, fever, and hypoxemia. Despite multiple intravenous antibiotics and frequent chest physiotherapy, the patient decompensates and is transferred to the intensive care unit for mechanical ventilation. Sputum cultures grow >100,000 colony-forming units of a non-lactose-fermenting gram-negative rod. After several days, the patient dies of overwhelming pneumonia complicated by respiratory failure. Since age 1, he had been hospitalized for recurrent episodes of multi-lobar pneumonia, sinusitis, and poor growth. On autopsy, both vas deferens are absent. Which of the following findings is also likely to be present on autopsy?

Distention and obstruction of pancreatic ducts The most common gastrointestinal disorder in patients with cystic fibrosis is pancreatic insufficiency. Mutations in the cystic fibrosis transmembrane conductance regulator gene lead to thick, viscous secretions in the lumens of the pancreas, resulting in obstruction, inflammation, and subsequent fibrosis. Clinical manifestations include steatorrhea, failure to thrive, and deficiency of fat-soluble vitamins.

Researchers studying gastrointestinal pathophysiology analyze hundreds of gastric mucosal biopsy specimens taken from patients who underwent endoscopy at a local tertiary care center. They notice that colonization of the gastric antrum with S-shaped, gram-negative bacteria is associated with a decreased number of somatostatin-producing antral cells. Depletion of these cellsfrom the gastric antrum is most likely to cause which of the following conditions?

Duodenal ulcer Helicobacter pylori antral gastritis is associated with the formation of duodenal ulcers due to increased gastric acid production. This increase in acidity is caused by unchecked gastrin production due to the destruction of somatostatin-secreting cells in the gastric antrum

A 6-year-old girl is brought to the office due to intermittent abdominal cramps, bloating, and diarrhea for 2 months. She also has excessive flatulence, which typically occurs after meals. The patient has had no fever or vomiting. She eats a well-balanced diet and takes no medications or vitamins. Vital signs are appropriate for age. Examination shows a mildly distended abdomen with no hepatosplenomegaly or tenderness. Laboratory evaluation reveals elevated tissue transglutaminase antibodies. Which of the following locations should be biopsied to confirm the diagnosis in this patient?

Duodenum Celiac disease is caused by an immune-mediated reaction to gluten and classically leads to chronic gastrointestinal symptoms. Diagnosis is confirmed by elevated tissue transglutaminase IgA antibody levels and duodenal biopsy showing villus flattening, intraepithelial lymphocyte infiltration, and crypt hyperplasia. Decreased mucosal absorption primarily affects Ñ Dital duodenum and/or proximal jejunum

A 55-year-old man comes to the office due to recurrent abdominal discomfort. He also has bloating, a feeling of fullness, and indigestion. Vital signs are within normal limits, and abdominal examination is unremarkable. Fecal occult blood testing is positive. Gastrointestinal endoscopic procedures are performed and confirm the presence of an ulcer. The patient is told that the ulcer itself is very unlikely to be malignant based on its location. Which of the following is the most likely site of this patient's gastrointestinal ulcer?

Duodenum Duodenal ulcers are not associated with an increased risk of carcinoma in the same location. In contrast, ulcers located in the esophagus, stomach (gastric), and colon may be malignant, and biopsy is required

A 56-year-old man comes to the office for upper abdominal pain occurring with meals for the last month. His symptoms improve when fasting. The patient's other medical problems are coronary artery disease and osteoarthritis. His medications include aspirin, simvastatin, and acetaminophen as needed. The patient drinks 1 or 2 beers daily but does not use tobacco or illicit drugs. Upper endoscopy is performed and reveals a gastric ulcer with a smooth, rounded border and an exudative base. A biopsy of the ulcer is taken and placed into a urea-containing solution that includes phenol red, a pH indicator. Thirty minutes later, the solution has turned pink, indicating a pH increase. This test result suggests that which of the following processes is occurring?

Enzyatic production of Ammonia Helicobacter pylori produces the enzyme urease, which splits urea into CO2 and ammonia and neutralizes the local acidic gastric pH. Active infection can be confirmed through rapid urease testing, in which gastric mucosa is evaluated in the presence of urea and a pH indicator. Evidence of alkalization (due to ammonia formation) is confirmatory.

A 1-month-old boy is brought to the office for follow-up of blood-streaked stools and diarrhea. The patient was born preterm at 35 weeks gestation and had no complications in the newborn nursery. He had been taking a standard cow's milk-based formula until 2 weeks ago when he developed loose stools streaked with blood and mucus.Symptoms resolved after changing the formula. If histopathologic examination had been performed when the patient was symptomatic, which of the following would most likely be seen?

Eosinophilic infirltration in the distal colon Food protein-induced allergic proctocolitis is a non-IgE-mediated reaction that causes inflammation and eosinophilic infiltration of the distal colon. Classic presentation is in early infancy with painless, blood-streaked stools that resolve with dietary avoidance of the offending food protein (eg, cow's milk).

A 70-year-old man comes to the office due to persistent epigastric discomfort and nausea for the past several months. He has also noticed black-colored stools on several occasions. Medical history is notable for hypertension and osteoarthritis. The patient emigrated from rural China 5 years ago to live with his daughter. He does not use tobacco, alcohol, or illicit drugs. Physical examination shows a thin male with pale mucous membranes, an enlarged left supraclavicular lymph node, and epigastric tenderness on deep palpation. Stool testing for occult blood is positive. Upper gastrointestinal endoscopy reveals a 3-cm ulceration at the gastric antrum, with a heaped-up irregular border. Biopsy from the edge of the ulcer reveals glandular structures containing intestinal-like columnar cells. Which of the following most likely predisposed this patient to his current condition?

Excessive consumption of salt-preserved food Intestinal-type gastric adenocarcinoma is visualized endoscopically as an ulcerated mass with irregular folded or heaped-up edges. Histologically, it resembles colon adenocarcinoma and is characterized by glandular structures containing intestinal-like columnar (or cuboidal) cells. Risk factors include Helicobacter pylori infection, a high-salt diet, n-nitroso-containing compounds, and autoimmune atrophic gastritis.

A 59-year-old woman comes to the office due to fatigue and generalized itching that have developed slowly over the last year. She has not had a visible rash. The patient is reluctant to seek health care and has not seen a physician in years. She does not use tobacco, alcohol, or illicit drugs. Vital signs are normal. Examination shows mild hepatomegaly. Laboratory studies show a striking increase in serum alkaline phosphatase, with moderate elevations in bilirubin and hepatic transaminases. Liver biopsy reveals dilated bile canaliculi with green-brown plugs and yellowish-green accumulations of pigment within the hepatic parenchyma. The patient does not return for her scheduled follow-up appointment to discuss further management. This patient is at risk for developing which of the following complications?

Fat soluble vitamin deficiency Cholestatic liver disease can cause malabsorption and nutritional deficiencies of fat-soluble vitamins

A 54-year-old man dies from profuse upper gastrointestinal hemorrhage. An autopsy is performed; gross examination of his liver is shown in the image below. This patient's liver findings most likely resulted from which of the following processes?

Fibrosis and nodular parenchymal regeneration Cirrhosis is characterized by diffuse hepatic fibrosis with replacement of the normal lobular architecture by fibrous-lined parenchymal nodules. Chronic viral hepatitis (eg, hepatitis B and C), alcohol, hemochromatosis, and nonalcoholic fatty liver disease are the most common causes of cirrhosis in the United States.

A 35-year-old woman comes to the clinic due to severe heartburn that is resistant to over-the-counter antacids. The patient has no known medical problems and takes no other medications. She occasionally has a glass of wine with dinner but does not use tobacco or illicit drugs. Physical examination shows scattered telangiectasias on the face, several ulcers at the tips of the fingers, and small calcium deposits in the soft tissues of the hands and elbows. Which of the following processes is the most likely cause of this patient's heartburn?

Fibrous replacement of the muscularis in the lower esophagus Systemic sclerosis may result in esophageal dysmotility and incompetence of the lower esophageal sphincter due to atrophy and fibrous replacement of the esophageal muscularis. This can cause gastroesophageal reflux with an increased risk of Barrett's esophagus and stricture formation.

34-year-old woman is found dead in her apartment. Medical history is significant for substance use disorder and a prior hospitalization for upper extremity cellulitis. An autopsy examination is performed. A ruptured plaque in the proximal anterior descending artery with an overlying occlusive thrombus is found. Incidentally, a 4-cm hepatic mass in the right lobe of the liver is also found. The mass is lighter than the surrounding liver tissue and appears lobulated with a central, gray-white, depressed stellate scar from which fibrous septae radiate to the periphery. The rest of the liver is normal. No additional abnormalities are noted on autopsy. Which of the following is the most likely diagnosis of this patient's liver lesion?

Focal nodular hyperplasia is a benign liver tumor marked by a central stellate scar containing an abnormally large artery. It usually arises in young women and most cases are asymptomatic and found incidentally.

A 68-year-old woman comes to the office for a preventive visit. The patient feels well, and recently lost 4 kg (8.8 lb) on a new diet and exercise regimen. She has no recent history of abdominal pain, jaundice, or changes in her stools. Past medical history is notable for hypertension and hypercholesterolemia, for which the patient takes appropriate medications. She does not use tobacco, alcohol, or illicit drugs. Vital signs are normal. On examination, the abdomen is soft and nontender, and no hepatosplenomegaly is noted. However, a firm mass is palpated in the right upper quadrant. CT scan of the abdomen is ordered and is shown below. This patient is at elevated risk for which of the following conditions?

Gallbladder adenocarcinoma Porcelain gallbladder is a potential manifestation of chronic cholecystitis and is often found in association with multiple gallstones. It is due to dystrophic intramural deposition of calcium salts in the setting of chronic inflammation. Porcelain gallbladder is associated with an increased risk of adenocarcinoma of the gallbladder.

A 46-year-old woman comes to the emergency department due to 3 days of persistent upper abdominal pain, nausea, and vomiting. The patient has had similar pain in the past, especially after fatty meals, which resolved spontaneously within several hours. Temperature is 38.6 C (101.5 F), blood pressure is 110/65 mm Hg, and pulse is 98/min. BMI is 33 kg/m2. Physical examination is notable for severe right upper quadrant tenderness. Leukocyte count is 21,000/mm3. Laparoscopic surgery is performed and reveals an erythematous, distended gallbladder with patchy necrosis. Which of the following events most likely initiated this patient's condition?

Gallbladder outflow obstruction Acute calculous cholecystitis is an acute inflammation of the gallbladder initiated by gallstone obstruction of the cystic duct. Subsequent steps in pathogenesis include mucosal disruption by lysolecithins, bile salt irritation of the luminal epithelium, prostaglandin release with transmural inflammation, gallbladder hypomotility, increased intraluminal pressure causing ischemia, and bacterial invasion.

A 34-year-old man comes to the office due to upper abdominal pain. The patient has vague discomfort that happens mostly in the afternoon and at night and is partially relieved by food. He sometimes feels nauseated. He has had no vomiting, black or bloody stools, or weight loss. The patient takes no prescription or over-the-counter medications. Family history is negative for cancer. Upper gastrointestinal endoscopy reveals a small ulcer with a clean base in the duodenal bulb. Biopsy of which of the following sites is most likely to demonstrate the infectious agent responsible for this patient's current condition?

Gastric antrum Helicobacter pylori is a common cause of peptic ulcers. Duodenal ulcers are associated with heavy colonization in the gastric antrum, whereas colonization in the gastric corpus is associated with gastric ulcers

A 36-year-old woman comes to the office due to vague epigastric abdominal pain, occasional nausea, and bloating. She has no associated weight loss. Medical history is notable for moderate obesity and chronic allergic rhinitis. The patient takes no medications and does not use tobacco, alcohol, or illicit drugs. She was a vegetarian for several years but now consumes a wide variety of foods, including meat products. The patient has traveled abroad extensively in the past. Vital signs are normal. Examination shows mild epigastric tenderness to deep palpation. Laboratory results reveal mild anemia. Upper gastrointestinal endoscopy shows diffuse erythema of the antral mucosa. Biopsy reveals an inflammatory cell infiltrate involving the superficial mucosal layers. The underlying cause of this patient's current condition can also lead to which of the following?

Gastric lymphoma Acute Helicobacter pylori infection initially causes nonatrophic antral gastritis and an increased risk for duodenal ulcers. Chronic infection results in patchy, multifocal, atrophic gastritis with loss of parietal cells and G cells in the gastric body; this is associated with decreased acid secretion and an increased risk of gastric ulcers, gastric adenocarcinoma, and MALT lymphoma.

A 32-year-old man comes to the clinic for peptic ulcer disease follow-up. The patient has received several months of proton pump inhibitor therapy without significant improvement in his epigastric discomfort. He does not use nonsteroidal anti-inflammatory drugs, tobacco, or alcohol. Helicobacter pylori testing is negative. The patient undergoes a partial gastrectomy for refractory peptic ulcer disease. The pathologist receives the tissue and notes significant enlargement of the gastric rugal folds on gross examination. Microscopy of the gastric mucosa reveals parietal cell hyperplasia. Which of the following stimuli is the most likely cause of parietal cell proliferation in this patient?

Gastrin Patients with Zollinger-Ellison syndrome develop peptic ulcer disease and parietal cell hyperplasia with gastric fold enlargement due to gastrin hypersecretion.

A 25-year-old woman comes to the office due to progressive fatigue and dizziness for several months. She also has intermittent loose stools and abdominal cramps but no hematochezia, melena, vomiting, or abnormal menses. Vital signs are within normal limits. Conjunctival pallor is present. The abdomen is mildly distended but nontender. Laboratory evaluation reveals anemia and a low ferritin level. Fecal occult blood testing is negative. An upper endoscopy is performed and reveals no visible abnormalities of the stomach or duodenum, although biopsies of the duodenum are significant for intraepithelial lymphocytosis. Which of the following is the most likely cause of this patient's condition?

HLA-Associated Autoimmune disorder Celiac disease is an autoimmune disorder triggered by dietary gluten that develops almost exclusively in patients with HLA-DQ2 or -DQ8 serotypes. It often causes diarrhea and malabsorption (eg, iron deficiency anemia). The earliest histologic finding is duodenal intraepithelial lymphocytosis; crypt hyperplasia and villous blunting develop later.

A 65-year-old man comes to the emergency department due to abdominal pain and diarrhea. Three weeks ago, he drove from Texas to Mexico for a family vacation. Temperature is 38.3 C (101 F), blood pressure is 115/70 mm Hg, and pulse is 98/min. Abdominal examination shows mild, generalized tenderness with no rebound tenderness or guarding. Leukocyte count is 14,000/mm3. Sigmoidoscopy demonstrates white-yellow plaques on the colonic mucosa, and biopsy shows that the plaques are composed of fibrin and inflammatory cells. Further questioning regarding this patient's trip to Mexico is most likely to reveal which of the following events?

He required hospitalization and Antibiotics for pneumonia Antibiotics disrupt the normal intestinal flora and which can allow for overgrowth of Clostridium difficile, an anaerobic, gram-positive, spore-forming bacillus. C difficile produces 2 toxins that penetrate colonic epithelial cells leading to watery diarrhea, abdominal cramping, and colitis. The presence of a pseudomembrane (exudate on colonic mucosa consisting of fibrin and inflammatory cells) is highly suggestive of C difficile infection.

A70-year-old man comes to the office due to abdominal pain that began 1 month ago. He has also experienced fatigue and weight loss. He has no chronic medical conditions and takes no daily medications. The patient worked in a factory that produced polyvinyl chloride for several decades and is now retired. Physical examination shows hepatomegaly. Imaging demonstrates a liver mass, which is subsequently resected. Immunohistochemical staining of the tumor cells is positive for CD31. Which of the following is the most likely diagnosis?

Hepatic angiosarcoma is an aggressive vascular malignancy associated with exposure to carcinogens (eg, vinyl chloride). The tumor cells express CD31 (ie, platelet endothelial cell adhesion molecule [PECAM1]), an endothelial cell marker important for leukocyte transmigration

A 32-year-old man comes to the clinic due to several months of fatigue and weight loss. The patient has no significant medical history. He drinks 1 or 2 alcoholic beverages daily and has used intravenous recreational drugs in the past. He is sexually active with his girlfriend. The patient has a maternal aunt with hypothyroidism, but his family history is otherwise unremarkable. Temperature is 36.7 C (98 F), blood pressure is 110/70 mm Hg, pulse is 65/min, and respirations are 18/min. Physical examination is normal. A liver biopsy is obtained, and light microscopy reveals large hepatocytes filled with finely granular, homogeneous, pale pink cytoplasm. Which of the following is the most likely diagnosis?

Hepatitis B infection causes the hepatocellular cytoplasm to fill with hepatitis B surface antigen. These inclusions are highly specific for hepatitis B infection and have a finely granular, pale eosinophilic, ground-glass appearance.

A 34-year-old man comes to the office due to 2 days of anorexia, nausea, and dark-colored urine.He travels frequently for business and recently returned from a trip to Mexico. The patient has a history of asthma and occasionally uses inhaled albuterol for symptom management. His temperature is 37.8 C (100.2 F), blood pressure is 125/70 mm Hg, and pulse is 94/min. Cardiopulmonary examination is normal. There is mild right upper quadrant abdominal tenderness with no guarding or rebound. Serologic testing for Entamoeba histolytica is negative. Which of the following findings is most likely to be seen on liver biopsy in this patient?

Hepatocyte swelling Acute hepatitis due to most hepatotropic viruses causes hepatocyte ballooning degeneration and apoptosis with mononuclear cell infiltration.

A 5-week-old boy is brought to the emergency department after having tonic-clonic movements of his left upper and lower extremity that lasted approximately 3 minutes. The patient was born at home at 38 weeks gestation to a 23-year-old primigravida. The patient's mother received prenatal care, took prenatal vitamins, and had a healthy diet during pregnancy. The infant's newborn screen was positive for cystic fibrosis, and confirmatory testing is pending. The infant has been breastfeeding and gaining weight well. He has never taken any medications and also has received no vaccinations. On examination, the infant is postictal but no longer seizing. A head CT scan shows a right-sided intracranial hemorrhage. Which of the following is the most likely cause of this infant's presentation?

Impaired gamma carboxylation Vitamin K is necessary for the carboxylation and functionality of coagulation factors II, VII, IX, and X. Newborns who do not receive prophylactic supplementation are at risk for bleeding complications. Patients with cystic fibrosis are also at risk for vitamin K deficiency due to poor absorption of fat-soluble vitamins.

A 32-year-old man comes to the office due to intermittent dysphagia for solids and liquids. He has no significant past medical history and does not use tobacco, alcohol, or illicit drugs. Hisfather has a history of esophageal squamous cell carcinoma. Physical examination is unremarkable. The patient undergoes an esophageal manometric study that demonstrates periodic, simultaneous, and non-peristaltic contractions of large amplitude and long duration. Which of the following is the most likely pathogenesis of this patient's esophageal condition?

Impaired neural inhibition Diffuse esophageal spasm is characterized by periodic, simultaneous, and non-peristaltic contractions of the esophagus due to impaired inhibitory innervation within the esophageal myenteric plexus. Patients typically present with liquid/solid dysphagia and chest pain due to inefficient propulsion of food into the stomach.

A 40-year-old woman comes to the office due to recurrent episodes of postprandial abdominal pain. The pain is in the right upper quadrant and is associated with nausea. The patient has no fever, jaundice, or loss of appetite. Other medical conditions include obstructive sleep apnea and obesity. The patient takes no medications. She has never had any surgeries. She does not use tobacco, alcohol, or illicit drugs. The patient is afebrile. Blood pressure is 140/90 mm Hg. BMI is 38 kg/m2. Examination shows tenderness in the right upper quadrant. Abdominal ultrasound demonstrates multiple gallstones. A gross specimen obtained from laparoscopic cholecystectomy is shown in the image below: Which of the following is the most likely mechanism responsible for gallstone formation in this patient?

Increased cholesterol synthesis Most gallstones occur due to the supersaturation of cholesterol, which precipitates out of bile with mucin and calcium salts to form white or yellow cholesterol stones. Factors that promote the formation of cholesterol stones include increased cholesterol synthesis, gallbladder hypomotility, increased calcium or mucin concentration, and decreased bile acid synthesis or recirculation.

A 31-year-old woman comes to the emergency department due to worsening right upper quadrant pain, fever, nausea, and vomiting for a day. The pain increases with deep inspiration. Other medical conditions include uterine fibroids and sickle cell anemia. The patient does not use tobacco, alcohol, or illicit drugs. Temperature is 38.6 C (101.5 F), blood pressure is 120/70 mm Hg, and pulse is 102/min. BMI is 24 kg/m2. On examination, the patient appears uncomfortable because of the pain; there is no jaundice. Tenderness and guarding are present over the right upper quadrant. Bowel sounds are decreased. Hemoglobin is 10.1 g/dL and white blood cell count is 18,000/mm3 with 7% band forms. Abdominal ultrasound shows evidence of cholelithiasis and gallbladder wall thickening. Gross inspection of the specimen obtained during laparoscopic cholecystectomy is shown in the image below: Which of the following is most likely responsible for this patient's symptoms?

Increased efflux of Bilirubin to bile Black gallstones form due to supersaturation of the bile with bilirubin, which precipitates with calcium to form calcium bilirubinate stones. This typically occurs in the setting of chronic hemolysis (eg, sickle cell disease) or altered enterohepatic circulation of bilirubin (eg, Crohn disease, ileal resection).

A 59-year-old man is evaluated in the clinic for recent unintentional weight loss. He says, "My pants are fitting a lot looser than usual." The patient also has epigastric discomfort after meals with occasional nausea. He reports that food is not as appetizing as it used to be. He has no known medical conditions and takes no medications. The patient's father has a history of peptic ulcer disease. Upper endoscopy shows a lesion in the stomach that is biopsied; a histologic section is shown below: This patient's lesion is most likely to demonstrate which of the following gross findings?

Infiltrative wall within the stomach wall There are 2 morphological variants of gastric adenocarcinoma. The intestinal type forms a solid mass that projects into the stomach lumen and is composed of gland-forming cuboidal or columnar cells. In contrast, diffuse-type adenocarcinoma infiltrates the stomach wall (linitis plastica) and displays signet ring cells on light microscopy.

A 75-year-old hospitalized man is evaluated for abdominal pain and fever. The patient was admitted 2 weeks ago due to necrotizing fasciitis of the gluteal region and severe sepsis. He has been treated with surgical debridements, intravenous antibiotics, and fluids. For the past 2 days, the patient has had right upper quadrant abdominal pain,nausea, and recurrent fever. He has a history of hypertension, type 2 diabetes mellitus, and chronic obstructive pulmonary disease. His temperature is 38.8 C (102 F). Physical examination shows marked tenderness over the right subcostal area. Leukocyte count is 18,000/mm3 compared to 9,600/mm3 2 days ago. Laparoscopic surgery is planned. Which of the following will most likely be found during this patient's surgery?

Inflammed and enlarged gallbladder Acute acalculous cholecystitis is an acute inflammation of the gallbladder in the absence of gallstones. It typically occurs in critically ill patients (eg, those with sepsis, severe burns, trauma, immunosuppression) due to gallbladder stasis and ischemia. Clinical findings may be subtle and include fever, right upper quadrant pain, and leukocytosis.

A 54-year-old man is evaluated in the clinic due to generalized weakness and lethargy for the past 5 years. He has no history of hypothyroidism or depression. The patient uses acetaminophen intermittently for joint pains that he attributes to "old age." He drinks alcohol occasionally but does not use tobacco or illicit drugs. His older brother died of liver cirrhosis. Laboratory tests show a serum ferritin level of 1800 μg/L. If this patient's disorder is hereditary, the genetic defect responsible for his condition most likely affects which of the following processes?

Intestinal Iron absorption Hereditary hemochromatosis is most commonly caused by a missense mutation in the HFE gene, resulting in excessive intestinal iron absorption and organ damage (eg, cirrhosis, diabetes mellitus, cardiomyopathy, arthropathy) due to iron accumulation within parenchymal tissues.

A 2-year-old Caucasian boy is hospitalized with abdominal pain and vomiting. After initial evaluation laparotomy is performed, and a portion of the intestine is resected (see the slide below). This patient most likely suffers from:

Intussusception most often occurs in children younger than 2 years of age and in the region of the ileocecal valve. It manifests with intermittent, severe, colicky abdominal pain, "currant jelly" stools, and sometimes a palpable mass in the right lower abdominal quadrant.

A mass resected from the liver of a 32-year-old woman has the morphology demonstrated on the slide below. Which of the following is the most accurate statement concerning this patient's condition?

It is the most common benign liver tumor Cavernous hemangioma is the most common benign liver tumor. Microscopically, these tumors consist of cavernous, blood-filled vascular spaces of variable size lined by a single epithelial layer. The biopsy of a suspected hemangioma is not advisable, as the procedure has been known to cause fatal hemorrhage and is of low diagnostic yield.

A 46-year-old man comes to the physician complaining of intermittent, bloody diarrhea and abdominal pain for the past month. He has lost 12 lbs (5.5 kg) during this period. He was diagnosed with HIV five years ago, but has been noncompliant with his antiretroviral medications. Laboratory results show a CD4 count of 50 cells/µL and viral load of 650,000 copies/mL. Colonoscopy reveals multiple hemorrhagic polypoidal lesions. Biopsy of these lesions reveals spindle cells with surrounding blood vessel proliferation. Which of the following is the most likely cause of this patient's diarrhea?

Kaposi's sarcoma usually involves the skin and GI tract and is common in HIV patients not on antiretroviral therapy. Endoscopy reveals characteristic lesions, which range from reddish/violet flat maculopapular lesions to raised hemorrhagic nodules or polypoid masses. Biopsy can show spindle cells, neovascularization, and extravasated red blood cells.

A 65-year-old man comes to the office with dysphagia. For the last 3 months, he has relied on protein shakes and juices for nourishment as he has had difficulty swallowing solid foods. He has also had a diminished appetite and an estimated weight loss of 9.1 kg (20 lb). Medical history includes hyperlipidemia, emphysema, and previous episodes of pancreatitis. Family history includes lung cancer in his mother. The patient smoked a pack of cigarettes daily for 50 years but quit 2 years ago. He has been drinking a fifth (750 mL) of vodka weekly for the last decade. Vital signs are normal. BMI is 19.2 kg/m2. Temporal wasting is present but physical examination is otherwise unremarkable. Upper endoscopy reveals a 4-cm mass in the middle third of the esophagus, and several biopsies are taken. Thoracic imaging reveals mediastinal lymphadenopathy. Histologic evaluation of the biopsy specimens will most likely show which of the following in this patient?

Keratin pearls and intercellular bridges Esophageal squamous cell carcinoma presents mostly in the proximal two-thirds of the esophagus and frequently metastasizes to the mediastinal lymph nodes. Histology characteristically shows sheets of atypical squamous cells with keratin pearls and intercellular bridges.

A 22-year-old Caucasian male presents to the ER with abdominal pain that started around his bellybutton, then moved to the right lower abdominal quadrant. After initial evaluation, laparotomy is performed, which reveals the following findings (see the slide below). Which of the following most likely initiated this patient's condition?

Lumen Obstruction Obstruction of the lumen of the appendix is the first event in pathogenesis of acute appendicitis. Fecaliths, hyperplastic lymphoid follicles, foreign bodies, or tumors may cause the obstruction. Right lower quadrant abdominal pain, nausea, vomiting, diarrhea, and fever are the typical manifestations of acute appendicitis.

A 65-year-old woman has had several months of progressive fatigue, diminished appetite, and unintentional weight loss. The patient has no chronic medical conditions. She has not received medical care in many years and has not had recommended cancer screening. The patient suddenly dies while undergoing evaluation. Autopsy reveals abnormal liver findings, as shown in the exhibit below. Microscopic evaluation of the lesions reveals neoplastic cells. Which of the following is the most likely cause of this patient's liver lesions?

Malignancy spreading through portal circulation Most hepatic neoplasms are due to metastatic disease from a distant site. Colorectal cancer is the most common cause of hepatic metastases due to direct blood flow from the colon (and superior rectum) to the liver via the portal venous circulation.

A 63-year-old man comes to the office due to several months of dysphagia, fatigue, occasional chest discomfort, and indigestion. He has a history of hypertension and takes amlodipine. The patient smokes 1 or 2 cigars daily. His blood pressure is 140/80 mm Hg. Physical examination is normal. An endoscopic evaluation with esophageal biopsy is performed. The histopathology of the specimen is shown in the image below. Which of the following is the most likely cause of this patient's dysphagia?

Malignancy with probable porr prognosis The histopathologic features of esophageal squamous cell carcinoma include solid nests of neoplastic squamous cells with abundant eosinophilic cytoplasm and distinct borders. Areas of keratinization and the presence of intercellular bridges are also characteristic. Patients typically present with progressive solid and eventually liquid dysphagia and weight loss.

A 53-year-old man comes to the physician with progressively worsening anorexia and abdominal discomfort. He has lost 14 kg (31 lb) since the onset of his symptoms about 4 months ago. Physical examination demonstrates nontender hepatomegaly. Laboratory studies show an elevated serum alkaline phosphatase level and a marginally elevated alanine aminotransferase level. A CT scan of the abdomen with oral and intravenous contrast is shown below.Which of the following is the most likely diagnosis?

Malignant hepatic lesions most often represent metastasis from another primary site (eg, breast, lung, colon); primary liver neoplasms (eg, hepatocellular carcinoma) are much less common

A 35-year-old man with a history of alcohol use disorder is brought to the ER with hematemesis. Prior to this, he has had three episodes of vomiting. His BP is 110/80 mmHg and pulse is 98/min. He has no jaundice. Lungs are clear to auscultation. Abdomen is soft, non tender, and non-distended. There is no hepatomegaly. Endoscopy shows longitudinal mucosal tears at the gastroesophageal junction. This patient's condition is most likely related to which of the following?

Mallory-Weiss tears account for about 10% of cases of upper GI hemorrhage. They occur due to increased intraluminal gastric pressure due to retching, vomiting, or other abdominal straining.

A 2-year-old girl is brought to the emergency department with intermittent pain in the abdomen that began several hours ago. The patient also had a bowel movement that appeared tinged with blood. Ultrasound shows an area along the ileum in which the proximal small intestine is telescoped into the distal small intestine. Reduction via enema is unsuccessful, and the patient undergoes laparotomy with resultant resection of the involved intestine. The pathologic finding at the lead point is shown in the following image: Which of the following is the most likely underlying diagnosis in this patient?

Meckel diverticulum Meckel diverticulum is the most common pathologic lead point for intussusception, a condition in which a portion of the intestine telescopes into itself. Meckel diverticula frequently contain ectopic gastric mucosa, which can aid in diagnosis.

A 69-year-old man is evaluated for abdominal pain. Upper gastrointestinal endoscopy shows a single shallow, 1-cm ulcer in the duodenal bulb and mild gastric erythema. Biopsy specimens from the gastric antrum do not show Helicobacter pylori, and stool testing for H pylori antigen is negative. Which of the following parts of thehistory is most likely to reveal the cause of this patient's current condition?

Medication use Peptic ulcers are most commonly caused by Helicobacter pylori infection or nonsteroidal anti-inflammatory drug (NSAID) use. NSAIDs inhibit cyclo-oxygenase-1, leading to increased gastric acid formation, decreased mucosal bicarbonate production, and decreased mucosal blood flow, which promote mucosal injury and ulcer formation.

A 5-year-old boy is brought to the emergency department due to diarrhea and a low-grade fever. He attends a day care center where other children developed similar symptoms. The boy has no known medical problems. He is up to date with age-appropriate vaccinations. He has not traveled abroad. Physical examination shows abdominal tenderness on deep palpation but no rigidity. Stool is sent for culture, and the patient is diagnosed with Shigella sonnei infection. The outbreak is reported to public health authorities and is traced to consumption of a potato salad handled by a food worker who recently had a mild diarrheal illness. Which of the following cells are primarily responsible for uptake of the bacteria that initiated the infection in this boy?

Microfold (M) cells Shigellosis is an infectious disease that can be caused by variety of Shigella species; S sonnei is the most common cause of shigellosis in industrialized nations. Shigella invades the gastrointestinal mucosa by gaining access to microfold cells in ileal Peyer patches through endocytosis. Shigella subsequently lyses the endosome and spreads laterally into other epithelial cells, causing cell death and ulceration with hemorrhage and diarrhea

A 6-year-old boy with no chronic medical conditions is brought to the emergency department due to confusion and intractable vomiting. His symptoms began a few hours ago and have gradually worsened. The patient's mother reports that he contracted a febrile illness from his younger brother 2 days ago. She gave the patient one tablet of an over-the-counter cold medication yesterday, and his fever resolved. While the mother is speaking, the patient lapses into a coma. If a liver biopsy were performed, light microscopy of the specimen would most likely reveal which of the following?

Microvesicular steatosis Reye syndrome is characterized by liver failure and encephalopathy following salicylate (aspirin) administration in children with a febrile illness. Aspirin-induced mitochondrial dysfunction causes impaired fatty acid metabolism and microvesicular steatosis of the liver.

A 68-year-old man comes to the office due to fatigue, weight loss, and constipation. He has refused to undergo screening colonoscopy for several years because "nobody in my family had colon cancers." Past medical history is notable for emphysema and stable angina. The patient has smoked a pack of cigarettes daily for 50 years and drinks alcohol occasionally. Physical examination is unremarkable. He is referred for colonoscopy, which reveals a circumferential mass encircling the sigmoid colon. Pathology is positive for adenocarcinoma. CT scan of the chest, abdomen, and pelvis is negative for signs of metastasis. Serum carcinoembryonic antigen assay is ordered. The results of this assay would be most useful for which of the following aspects of this patient's care?

Monitoring for residual disease Carcinoembryonic antigen (CEA) levels are increased in colon cancer but are also elevated in a number of other conditions (eg, pancreatic cancer, COPD, cirrhosis). CEA cannot be used to diagnose colon cancer, but it is helpful for detecting residual disease and recurrence

A 33-year-old woman with a 9-year history of ulcerative colitis comes to the office due to concern about her risk of colon cancer. One of her close family friends died recently from this cancer. The patient wants to "undergo all the tests for colon cancer." She has no other comorbid conditions and is under the care of a gastroenterologist who has kept her disease well controlled with sulfasalazine. Compared with sporadic colorectal carcinoma,colorectal malignancy arising in this patient would more likely exhibit which of the following features?

Multifocal origin Inflammatory bowel disease, especially ulcerative pancolitis, is associated with a significant risk for colorectal carcinoma. Compared with sporadic colorectal cancer, colitis-associated colorectal cancer is more likely to occur at a younger age, is typically more aggressive with a higher histopathologic grade, often evolves from flat (nonpolypoid) lesions, and is frequently multifocal. Patients should be monitored regularly via colonoscopy with random biopsies.

A 55-year-old man comes to the office due to intermittent abdominal pain. He has no difficulty or pain with swallowing. The patient occasionally uses over-the-counter pain medicine for osteoarthritis of the right knee. He does not use alcohol or tobacco and has no history of cancer in the family. He works as an air traffic controller at a busy airport. Upper gastrointestinal endoscopy shows an ulcer in the distal duodenum. Fasting serum gastrin concentration is at the upper limit of normal and rises in response to intravenous secretin. This patient's condition is most likely caused by which of the following processes?

Neoplasm Zollinger-Ellison syndrome is caused by gastrinomas located in the small intestine/pancreas and presents with peptic ulcers (especially distal duodenal ulcers), heartburn, and diarrhea. Patients typically have elevated gastrin levels that rise in response to exogenous secretin administration. In contrast, secretin inhibits release of gastrin from normal gastric G cells.

A 65-year-old man with a history of chronic gastroesophageal reflux comes to the clinic due to dysphagia. The patient has had difficulty swallowing foods such as steak or hard-boiled eggs but is able to drink hot tea and coffee without any issues. Increasing the frequency of his proton pump inhibitor to twice daily did not improve symptoms. He has also lost 6.8 kg (15 lb) over the past 4 months. The patient has a history of hypertension, hyperlipidemia, and osteoarthritis. Medications include amlodipine, atorvastatin, and ibuprofen as needed. He drinks 2 or 3 glasses of wine with dinner most nights but does not use tobacco or illicit drugs. Vital signs are within normal limits. BMI is 32 kg/m2. Physical examination is unremarkable. Esophagogastroduodenoscopy reveals a 4-cm mass surrounded by a large segment of discolored mucosa in the lower half of the esophagus. Esophagogastrectomy results are shown in the image below: Which of the following is the most likely predisposing factor for this patient's esophageal disease?

Obesity Esophageal adenocarcinoma usually occurs in the distal esophagus due to underlying Barrett esophagus. Long-standing gastroesophageal reflux disease is the most important risk factor. Obesity, smoking, use of medications that lower esophageal sphincter pressure, and consumption of foods containing nitroso compounds also increase the risk.

A 46-year-old woman comes to the office due to dysphagia. The patient has had difficulty swallowing both liquids and solid foods as well as frequent episodes of regurgitation and cough while eating. She has also had weakness in her extremities and a rash. The patient has no prior medical problems and takes no medications. Vital signs are within normal limits. Physical examination shows an erythematouseruption on the upper eyelids. The oropharynx is clear and cardiopulmonary and abdominal examinations are unremarkable. She has difficulty lifting her arms above her head and standing from a sitting position. Which of the following is the most likely cause of this patient's dysphagia?

Perifascicular atrophy of muscle fibers in the proximal esophageal muscularis propria Dermatomyositis is characterized by proximal muscle weakness resembling polymyositis, with additional inflammatory features affecting the skin (eg, heliotrope rash, Gottron papules). In both conditions, involvement of striated muscle in the oropharynx and heart can lead to dysphagia, pulmonary aspiration, and myocarditis.

A 30-year-old man comes to the emergency department with a 4-day history of progressively worsening abdominal pain and bloody diarrhea. He was started on mesalamine therapy 6 months ago after being diagnosed with ulcerative colitis but has been noncompliant with treatment. His temperature is 38.8 C (102 F), blood pressure is 100/70 mm Hg, and pulse is 130/min. The patient is lethargic and has dry mucous membranes. There is marked abdominal distension and tenderness without rebound or guarding. Rectal examination shows guaiac-positive, maroon-colored, liquid stool. Which of the following is the best next step in this patient's workup?

Plain Abdominal X Ray Toxic megacolon is a well-recognized complication of ulcerative colitis. Patients typically present with abdominal pain/distension, bloody diarrhea, fever, and signs of shock. Plain abdominal x-ray is the preferred diagnostic imaging study. Barium contrast studies and colonoscopy are contraindicated due to the risk of perforation.

A 46-year-old man is brought to the emergency department by paramedics after an episode of large-volume hematemesis. He was going over some papers at work when he suddenly felt nauseated and started vomiting up bright red blood. Physical examination reveals a palpable spleen. Endoscopy shows bleeding esophageal varices. A liver biopsy performed 2 days later shows no abnormalities. Which of the following is the most likely cause of this patient's condition?

Portal vein thrombosis Portal vein thrombosis causes portal hypertension, splenomegaly, and varicosities at portocaval anastomoses. It does not cause histologic changes to the hepatic parenchyma. Ascites is uncommon as the obstruction is presinusoidal; ascites typically only develops in conditions that cause sinusoidal hypertension.

A 40-year-old man comes to the office with pain on defecation. The pain is sharp and severe but subsides within minutes of passing the bowel movement. He often sees spots of bright red blood on the toilet paper. The patient has chronic constipation and has not taken laxatives or increased his fluid intake to compensate. He has no fever, night sweats, diarrhea, or unexpected weight loss. Past medical history is notable for chronic low back pain following a motor vehicle accident, for which he takes a scheduled dose of pain relievers. The patient has smoked a pack of cigarettes daily for the last 20 years but does not use alcohol or illicit drugs. He is married and has no outside sexual contacts. Vital signs are normal. The abdomen is soft and nontender with no hepatosplenomegaly. Rectal examination is likely to show a tear at which of the following locations?

Posterior midline distal to pectinate line Anal fissures are longitudinal tears in the mucosa. They are usually due to passage of hard stool in patients with chronic constipation. Most fissures occur at the posterior midline, likely due to decreased blood flow in this area. Fissures in other areas may be due to less common causes.

A 45-year-old woman recently diagnosed with Sjögren's syndrome complains of severe pruritus that slowly progressed over the last year. She says that pruritus is especially prominent over palms and soles and isunbearable during the night. Laboratory testing shows: Total bilirubin1.0 mg/dL Direct bilirubin0.8 mg/dL Alkaline phosphatase720 U/L Aspartate aminotransferase (SGOT)48 U/L Alanine aminotransferase (SGPT)40 U/L This patient most likely has which of the following conditions?

Primary biliary cholangitis Primary biliary cholangitisPrimary biliary cholangitis is a chronic liver disease characterized by autoimmune destruction of the intrahepatic bile ducts and cholestasis (elevated alkaline phosphatase). The condition is most common in middle-aged women, with severe pruritus (especially at night) often one of the first reported symptoms

A 45-year-old woman comes to the office for progressive itchiness and fatigue. She also notes yellowing of the eyes and skin. Physical examination shows scleral icterus, multiple excoriations on both the upper and the lower extremities, and hepatomegaly. Laboratory results are as follows: Liver function studies Total bilirubin5.3 mg/dL Alkaline phosphatase982 U/L Aspartate aminotransferase (SGOT)89 U/L Alanine aminotransferase (SGPT)67 U/L Gamma-glutamyl transpeptidase450 U/L (normal: 10-60 U/L) Liver biopsy reveals dense lymphocytic infiltration of the portal triads, as well as granulomatous destruction of interlobular bile ducts. Which of the following is the most likely diagnosis?

Primary biliary cholangitis Primary biliary cholangitis is an autoimmune disorder that most commonly affects middle-aged women and is characterized by fatigue, pruritus, jaundice, cholestasis (eg, elevated alkaline phosphatase, elevated gamma-glutamyl transferase, direct hyperbilirubinemia), and antimitochondrial antibodies. Histologic findings include dense lymphocytic portal tract inflammation with granulomatous destruction of interlobular bile ducts

A 42-year-old man comes to the office due to progressive fatigue, nausea, and itchiness. On review of systems, the patient admits that he occasionally has bloody stools. Medical history is unremarkable. Examination shows scleral icterus. Percutaneous liver biopsy is performed, and histologic analysis of the tissue sample reveals fibrous obliteration of intrahepatic small bile ducts with concentric replacement by connective tissue. Which of the following is the most likely diagnosis?

Primary sclerosing cholangitis Primary sclerosing cholangitis is characterized by inflammation, fibrosis, and stricture of intrahepatic and extrahepatic bile ducts. Histologic findings include fibrous obliteration of bile ducts and concentric periductal connective tissue deposition (eg, onion skin-like pattern). Manifestations reflect cholestasis and include jaundice, pruritus, fatigue, and a cholestatic injury pattern (ie, elevated alkaline phosphatase, direct hyperbilirubinemia)

A 44-year-old man with a history of heavy alcohol consumption comes to the emergency department with a nosebleed after getting into a fight while intoxicated. Continuous local pressure is applied and his bleeding resolves within 30 minutes. A detailed physical examination is performed after the bleeding subsides and shows distended paraumbilical veins, ascites, and a flapping hand tremor on wrist extension. Which of the following laboratory findings would be most indicative of a poor prognosis for this patient?

Prolonged prothombin Time Increased aspartate aminotransferase and alanine aminotransferase are indicators of hepatocellular damage, and increased alkaline phosphatase and gamma-glutamyl transpeptidase indicate biliary injury. Serum albumin levels, bilirubin levels, and prothrombin time are reflective of liver function and are of greatest prognostic significance in patients with cirrhosis

A 72-year-old man develops mild abdominal pain and bloody diarrhea after undergoing urgent cholecystectomy. The surgery was complicated by an episode of hypotension that was treated with bolus intravenous crystalloid fluids. The patient had no previous gastrointestinal bleeding. His last colonoscopy 10 years ago was normal. Past medical history is notable for hypertension, hyperlipidemia, type 2 diabetes mellitus, and a myocardial infarction 7 years ago. The patient smoked for 40 years and quit after the myocardial infarction. Colonoscopy would most likely show pathology in which of the following portions of the large bowel?

Rectosigmoid junction The splenic flexure and rectosigmoid junction lie between regions of perfusion of major arteries. These "watershed" areas are susceptible to ischemic damage during hypotensive states, especially in patients with underlying arterial insufficiency.

A 54-year-old man comes to the emergency department after an episode of bloody vomiting. He has had no fever, chills, abdominal pain, diarrhea, or constipation. The patient has a history of alcohol abuse with multiple previous hospital admissions due to alcohol withdrawal and generalized tonic-clonic seizures. He also has a history of intravenous drug abuse and is currently enrolled in a methadone maintenance program. Blood pressure is 96/62 mm Hg and pulse is 102/min. On examination, the patient appears comfortable but develops dizziness when asked to sit up. There is no jugular venous distension. The abdomen is distended with dullness to percussion at both flanks. The liver is enlarged on palpation, and the tip of the spleen is also palpable. There is trace pedal edema. Laboratory studies show: Hemoglobin9.7 g/dL Mean corpuscular volume98 fL Leukocytes5,000/mm3 Platelets78,000/mm3 Histopathologic examination of this patient's spleen is most likely to show which of the following?

Red pulp expansion Portal hypertension, as seen in alcoholic liver disease, produces splenomegaly by causing congestion of blood within the spleen, which produces expansion of the red pulp.

A 55-year-old man undergoing evaluation for fatigue and exertional dyspnea is diagnosed with macrocytic anemia. Upper gastrointestinal endoscopy is consistent with atrophic gastritis. He is started on intramuscular cyanocobalamin, with the resulting changes shown in the graph below. The blue curve most likely corresponds to which of the following parameters?

Reticulocyte count Atrophic gastritis can result in profound hypochlorhydria, inadequate intrinsic factor production, vitamin B12 deficiency, and elevated methylmalonic acid levels. The reticulocyte count increases dramatically once vitamin B12 replacement therapy is initiated in an individual with pernicious anemia. Hemoglobin and erythrocyte count levels rise more gradually and take up to 8 weeks to normalize.

A 3-week-old male born to a 23-year-old Caucasian primigravida develops projectile non-bilious vomiting after every meal. Abdominal inspection reveals prominent peristalsis in the epigastrium and an olive-sized mass is felt on deep palpation of the right upper abdomen. The mass most likely represents:

Smooth muscle hypertrophy Congenital pyloric stenosis arises secondary to hypertrophy of the pyloric muscularis mucosae.

After initial stabilization with intravenous fluids and blood products, the patient develops another bout of bloody emesis. Urgent upper gastrointestinal endoscopy confirms esophageal varices. Which of the following peptides is most likely to immediately decrease the portal venous pressure in this patient?

Somatostatin Acute management of variceal hemorrhage requires rapid lowering of portal pressure. Somatostatin and octreotide (a long-acting somatostatin analog) inhibit the release of hormones that induce splanchnic vasodilation, indirectly causing splanchnic vasoconstriction and reduced portal blood flow

A 45-year-old man with a history of alcohol use disorder, chronic hepatitis C, and HIV is brought to the emergency department due to altered mental status and abdominal distension. He is disoriented and unable to provide an adequate history. He is accompanied by a friend who reports that the patient has recently been drinking heavily. The patient's breath has a sweet, sulfurous odor. Physical examination shows gynecomastia, palmar erythema, and multiple spider angiomata. The abdomen is severely distended, and dilated periumbilical veins are noted. There is 3+ bilateral lower extremity edema. Genital examination shows testicular atrophy. Neurologic examination shows disorientation and asterixis. Abdominal imaging reveals splenomegaly. Development of this patient's gynecomastia most closely resembles the pathogenesis of which of the following additional findings in this patient?

Spider angiomata In patients with cirrhosis, gynecomastia often occurs due to hyperestrinism, which can also promote the formation of spider angiomata.

A 73-year-old man comes to the clinic with epigastric pain that starts 30-40 minutes after meals and does not respond to antacids. The pain is non-radiating and is graded 6/10 in intensity. The patient has lost more than 4.5 kg (10 lb) over the last few months, which he attributes to eating less due to his fear of the pain. He has no vomiting, diarrhea, or urinary symptoms. His past medical history is significant for hypertension, hyperlipidemia, coronary artery bypass grafting, and right-sided carotid endarterectomy. He has smoked a pack of cigarettes daily for 32 years. Upper gastrointestinal endoscopy shows no abnormalities. The underlying pathophysiology of this patient's disease process is most similar to which of the following conditions?

Stebale angina Chronic mesenteric ischemia is characterized by atherosclerosis of the mesenteric arteries, resulting in diminished blood flow to the intestine after meals. This causes postprandial epigastric pain (intestinal angina) with associated food aversion/weight loss. Its pathogenesis is similar to angina pectoris.

A 45-year-old man with diabetes mellitus is admitted to the intensive care unit due to community-acquired pneumonia, septic shock, and respiratory failure. Two days ago, he developed fever, productive cough, and shortness of breath that has progressively worsened. The patient is intubated and placed on mechanical ventilation. He is also started on intravenous fluids, broad-spectrum antibiotics, and a norepinephrine infusion. On the third day of hospitalization, hemoglobin level drops from 14 g/dL to 12 g/dL. Nasogastric suctioning reveals bright red blood. This patient's bleeding is most likely a result of which of the following processes?

Stress-related mucosal Injury Stress-related mucosal disease is usually caused by local ischemia in the setting of severe physiologic stress (eg, shock, extensive burns, sepsis, severe trauma). Ulcers arising in the setting of severe trauma/burns are called Curling ulcers. Ulcers arising from intracranial injury are caused by direct vagal stimulation and are called Cushing ulcers

An 8-month-old boy is brought to clinic due to abdominal distension. The patient has had excessive straining and irritability with bowel movements since birth. He had been exclusively breastfed, and his symptoms worsened after transitioning to pureed foods. Vital signs are normal. Abdominal examination shows distension with mild tenderness to palpation. Bowel sounds are present, and there is no hepatosplenomegaly. Barium enema shows a relatively narrow rectum and rectosigmoid area. The rest of the colon proximal to this segment is significantly dilated. Colorectal biopsy of which of the following areas is most likely to reveal the underlying cause of this patient's condition?

Submucosa of the narrow segment Absence of the submucosal (Meissner) and myenteric (Auerbach) plexuses in Hirschsprung disease causes the affected rectosigmoid region to become narrowed. The submucosa of the narrowed area is the most superficial layer where the absence of ganglion cells can be confirmed during biopsy procedures.

A 32-year-old woman comes to the office due to postprandial abdominal pain and nausea. The patient has no diarrhea, constipation, bloody stools, or vomiting. Medical history is significant for rheumatoid arthritis. The abdomen is mildly tender to palpation at the epigastrium but without distension or hepatosplenomegaly. Laboratory studies reveal anemia. A small gastric ulcer is seen during upper endoscopy, and biopsies are negative for Helicobacter pylori. Biopsies of the duodenum reveal numerous intraepithelial lymphocytes, villous atrophy, and crypt hyperplasia. Based on these biopsy findings, this patient is at increased risk for developing which of the following conditions later in life?

T-cell Lymphoma Celiac disease, an immune-mediated disorder triggered by gluten, causes small intestinal intraepithelial lymphocytosis, villous atrophy, and crypt hyperplasia. Monoclonal T-cell expansion can occur in the small-bowel mucosa of patients, leading to enteropathy-associated T-cell lymphoma.

A 27-year-old womanundergoes laparotomy for right lower quadrant pain and is found to have an inflamed terminal ileum. The involved segment of the intestine is removed, and the lesion observed on light microscopy is seen in the image. Which of the following cell types most likely mediated the formation of this patient's lesion?

Th-1 cells Granulomas are characterized by a large number of epithelioid macrophages that may fuse together to form multinucleated cells (Langhans giant cells) surrounded by a band of lymphocytes. Granuloma formation involves chronic Th1 and macrophage activation in response to a difficult-to-eradicate antigen.

A 60-year-old man is found to have a 5-cm exophytic mass in the ascending colon during a screening colonoscopy. Biopsy of the mass shows adenocarcinoma. The patient has no personal or family history of malignancy or colonic polyps. His medical problems include hypertension, and he does not use tobacco or alcohol. CT scan of the chest and abdomen reveals no metastatic lesions in the lungs or liver. A right colectomy is performed. The presence of which of the following indicates a poor prognosis in this patient?

Tumor cells deposit in the regional Lymph nodes Tumor stage (degree of invasion/spread) is the most important prognostic determinant for colorectal cancer. Tumors confined to the basement membrane and lamina propria are early stage and have the best prognosis. Outcomes generally deteriorate as the stage advances: spread of tumor into the muscularis propria → regional lymph nodes → distant sites

A 55-year-old Caucasian male is found on colonoscopy to have a solitary mass in his sigmoid colon. Biopsy is consistent with colon cancer, and surgery is scheduled. Which of the following features would carry the worst prognosis in this patient?

Tumor penetration of the muscularis propia Prognosis of colorectal adenocarcinoma is directly related to the stage of the tumor (not to the grade!).

A 60-year-old woman comes to the office with persistent, profuse mucoid diarrhea. Despite not eating much for the past 2 days, her diarrhea has not decreased, but she has no other symptoms. The patient has not had age-appropriate colon cancer screening. Family history is unremarkable. She does not use tobacco, alcohol, or illicit drugs. Vital signs and cardiopulmonary examination are normal. The abdomen is soft and nondistended. There is no hepatosplenomegaly. Laboratory studies show hypokalemia and microcytic anemia. Colonoscopy reveals a 2.5-cm, cauliflower-like mass in the sigmoid colon. The mass is resected and the histopathology of the lesion is shown in the exhibit. Which of the following is the most likely diagnosis in this patient?

Villous adenoma Adenomatous polyps are either tubular, villous, or tubulovillous, depending on their histologic appearance. Villous adenomas tend to be larger, sessile, and more severely dysplastic than tubular adenomas. Villous adenomas can cause a secretory diarrhea from increased mucin production; patients may develop hypoproteinemia and hypokalemia.

A 15-year-old boy comes to the office for a follow-up appointment. Since early childhood, he has suffered from recurrent respiratory infections and chronic diarrhea. The patient has been hospitalized several times for parenteral antibiotic treatment. Current prescribed medications include pancreatic enzyme therapy and a number of dietary supplements. Physical examination shows decreased proprioception and hyporeflexia in the lower extremities. Laboratory findings are suggestive of mild hemolytic anemia. Which of the following conditions is the most likely cause of these findings?

Vitamin E deficiency can occur in individuals with fat malabsorption. Deficiency of this fat-soluble vitamin is associated with increased susceptibility of the neuronal and erythrocyte membranes to oxidative stress. Clinical manifestations include ataxia, impaired proprioception and vibratory sensation, and hemolytic anemia

A 65-year-old man is referred to a gastroenterology clinic by his primary care provider after he develops symptoms suspicious for colorectal cancer. The patient has a history of obesity, diabetes mellitus, and chronic obstructive pulmonary disease. He often eats steaks and hamburgers but avoids vegetables as "they have no flavor." The patient has smoked a pack of cigarettes daily for the past 40 years and occasionally consumes alcohol. He undergoes a colonoscopy and is found to have a mass lesion in the ascending colon. The mass is biopsied, and the histopathologic findings are consistent with colon adenocarcinoma. This patient most likely initially presented with which of the following clinical symptoms?

Weight loss and progressive fatige Right-sided colon cancers usually grow as exophytic masses and present with occult bleeding and symptoms of iron deficiency anemia. Left-sided colon cancers tend to infiltrate the intestinal wall and encircle the lumen, causing constipation and symptoms of intestinal obstruction. Rectosigmoid involvement often causes hematochezia.

A 4-day-old newborn is brought to the office for a 2-day history of progressive jaundice. He was born at term following an uncomplicated vaginal delivery. The patient has been breastfed since birth. Vital signs are normal. Examination shows scleral icterus and jaundice from the face to the upper abdomen. Laboratory results are as follows: Complete blood count Hematocrit52% Reticulocytes1% Liver function studies Total bilirubin10.3 mg/dL Direct bilirubin0.3 mg/dL Aspartate aminotransferase (SGOT)18 U/L Alanine aminotransferase (SGPT)24 U/L Which of the following is the most likely explanation for these findings?

decreased UDP glucuronosyltransferase Benign neonatal hyperbilirubinemia presents with jaundice and elevated indirect bilirubin levels that peak at age 2-5 days. Pathophysiology includes impaired bilirubin clearance by the liver due to decreased hepatic UDP glucuronosyltransferase, an enzyme responsible for bilirubin conjugation.

A 45-year-old man comes to the office due to several months of diarrhea. The patient experiences watery diarrhea shortly after eating,occasional abdominal discomfort, easy fatigability, and unintentional weight loss. He has a history of Crohn disease for which he takes mesalamine. The patient also takes prednisone intermittently for disease exacerbations, and 6 months ago he was treated with metronidazole for a perianal abscess. Surgical history includes several intestinal resections due to complications of Crohn disease. He does not use tobacco or illicit drugs but drinks alcohol occasionally. Physical examination shows conjunctival pallor. The abdomen is nondistended, soft, and nontender. Vibratory and position sense are decreased in the lower extremities. Stool guaiac test is negative. Laboratory studies show macrocytic anemia and a low vitamin B12 level. Which of the following is the most likely cause of this patient's presenting symptoms?

loss of intestinal absorptive area Short bowel syndrome typically occurs in patients with massive small bowel resection and/or Crohn disease due to loss of intestinal absorptive surface area and a decrease in intestinal transit time. Patients usually present with postprandial voluminous diarrhea and weight loss due to malabsorption. Loss of functional distal ileum may result in vitamin B12 deficiency.


Set pelajaran terkait

spanish oral questions and answers

View Set

Physical Geology - Chapter 5 Test Bank

View Set

Restrictive and Nonrestrictive Words, Phrases, and Clauses, Restrictive and Nonrestrictive Words, Phrases, and Clauses - English 4

View Set

The Art of Public Speaking: Chapter 9

View Set

Unit 5 Progress Check: MCQ Ap Art History

View Set

Targeted Medical-Surgical: Endocrine

View Set

Alternating-Current Circuits and Electromagnetic Waves (Chap. 21)

View Set